- Mon Oct 16, 2017 2:30 pm
#40572
Complete Question Explanation
(The complete setup for this game can be found here: lsat/viewtopic.php?t=14969)
The correct answer is (C)
If F and I are in a group, then that group also includes L (from the second rule where F and L are always together). As we then think about our distribution options, the 4-1-1-1 possibility is out: I must be a single in that distribution, with the 4-part group consisting of F, L, M, and H.
Instead, we must be in a 3-2-1-1 distribution, with the FLI group as the triple, K as a single, and the other three candidates—G, H, and M—split into a double and single, remembering of course that G always goes to U.
So we could potentially put H or M with G on U, or we could leave G single and put H and M together elsewhere.
In short, we still have quite a few options placement-wise (no surprise really, considering that each of the four wrong answers in this question could be true...i.e. there's a lot of possibility allowed for by the answers themselves).
So what do we know for sure then?
Well, the FLI block has to go to either R or S. Putting that 3-person block on T would violate the S > T rule (the last rule), and putting the 3-person block on U would group it with G and give us a 4-1-1-1 distribution, where S and T are both singles (also breaking the last rule).
And voila! The right answer, answer choice (C), describes that impossibility: putting L on U means the whole FLI block goes there with G, and that can't happen for the reason described in the last paragraph!
The other four answers are all allowed:
Answer choice (A): As I mention above, H and G can go together. That would leave M and K as singles (one of which is on T and the other on R). FLI would go with S.
Answer choice (B): K, our always-alone variable, can go to T without issue. The FLI block could then go to either R or S and be fine.
Answer choice (D): If M goes to S, then H has to go with it so that S can have 2 people and be greater than T. That means our singles are K on T, and G on U, and the three-person block FLI goes to R.
Answer choice (E): Like answer choice (A), M and G can go together. That simply makes H and K our singles, one of which, again, is assigned to T and the other assigned to R. FLI would be assigned to S in this scenario.
(The complete setup for this game can be found here: lsat/viewtopic.php?t=14969)
The correct answer is (C)
If F and I are in a group, then that group also includes L (from the second rule where F and L are always together). As we then think about our distribution options, the 4-1-1-1 possibility is out: I must be a single in that distribution, with the 4-part group consisting of F, L, M, and H.
Instead, we must be in a 3-2-1-1 distribution, with the FLI group as the triple, K as a single, and the other three candidates—G, H, and M—split into a double and single, remembering of course that G always goes to U.
So we could potentially put H or M with G on U, or we could leave G single and put H and M together elsewhere.
In short, we still have quite a few options placement-wise (no surprise really, considering that each of the four wrong answers in this question could be true...i.e. there's a lot of possibility allowed for by the answers themselves).
So what do we know for sure then?
Well, the FLI block has to go to either R or S. Putting that 3-person block on T would violate the S > T rule (the last rule), and putting the 3-person block on U would group it with G and give us a 4-1-1-1 distribution, where S and T are both singles (also breaking the last rule).
And voila! The right answer, answer choice (C), describes that impossibility: putting L on U means the whole FLI block goes there with G, and that can't happen for the reason described in the last paragraph!
The other four answers are all allowed:
Answer choice (A): As I mention above, H and G can go together. That would leave M and K as singles (one of which is on T and the other on R). FLI would go with S.
Answer choice (B): K, our always-alone variable, can go to T without issue. The FLI block could then go to either R or S and be fine.
Answer choice (D): If M goes to S, then H has to go with it so that S can have 2 people and be greater than T. That means our singles are K on T, and G on U, and the three-person block FLI goes to R.
Answer choice (E): Like answer choice (A), M and G can go together. That simply makes H and K our singles, one of which, again, is assigned to T and the other assigned to R. FLI would be assigned to S in this scenario.
Jon Denning
PowerScore Test Preparation
Follow me on Twitter at https://twitter.com/jonmdenning
My LSAT Articles: http://blog.powerscore.com/lsat/author/jon-denning
PowerScore Test Preparation
Follow me on Twitter at https://twitter.com/jonmdenning
My LSAT Articles: http://blog.powerscore.com/lsat/author/jon-denning